If you find any mistakes, please make a comment! Thank you.

Chapter 10 Exercise A


1. Solution: If $T$ is invertible, then there exists $S\in\ca L(V)$ such that $TS=ST=I$. Then it follows from 10.4 that\[\ca M(S,(v_1,\cdots,v_n))\ca M(T,(v_1,\cdots,v_n))=\ca M(ST,(v_1,\cdots,v_n))=I\]\[\ca M(T,(v_1,\cdots,v_n))\ca M(S,(v_1,\cdots,v_n))=\ca M(TS,(v_1,\cdots,v_n))=I.\]Hence $\ca M(T,(v_1,\cdots,v_n))$ is invertible.

If $\ca M(T,(v_1,\cdots,v_n))$ is invertible, then there exists $B\in \mb F^{n,n}$ such that \[B\ca M(T,(v_1,\cdots,v_n))=\ca M(T,(v_1,\cdots,v_n))B.\]Note that by 3.60, we have an isomorphism between $\ca L(V)$ and $\mb F^{(n,n)}$ by taking $W=V$ and $w_i=v_i$ for all $i$, we can choose $S\in\ca L(V)$ such that $B=\ca M(S,(v_1,\cdots,v_n))$. Again by 10.4, we have\[\ca M(ST,(v_1,\cdots,v_n))=B\ca M(T,(v_1,\cdots,v_n))=I,\]\[\ca M(TS,(v_1,\cdots,v_n))=\ca M(T,(v_1,\cdots,v_n))B=I.\]Therefore $ST=I$ and $TS=I$, hence $T$ is invertible.

See also Linear Algebra Done Right Solution Manual Chapter 10 Problem 1.


2. Solution: Let $V$ be $\mb F^{n}$, where $n$ is the number of rows of $A$. Fix a basis $v_1,\cdots,v_n$ of $V$. By 3.60, we choose $T,S\in\ca L(V)$ such that\[\ca M(T,(v_1,\cdots,v_n))=A\quad\text{and}\quad\ca M(S,(v_1,\cdots,v_n))=B.\]Since $AB=I$, it follows from 10.4 that\[\ca M(TS,(v_1,\cdots,v_n))=\ca M(T,(v_1,\cdots,v_n))\ca M(S,(v_1,\cdots,v_n))=AB=I,\]hence $TS=I$. By Problem 10 of Exercise 3D, we have $ST=I$. Again by 10.4, we have\[BA=\ca M(S,(v_1,\cdots,v_n))\ca M(T,(v_1,\cdots,v_n))=\ca M(ST,(v_1,\cdots,v_n))=I.\]See also Linear Algebra Done Right Solution Manual Chapter 10 Problem 2.


3. Solution: See Linear Algebra Done Right Solution Manual Chapter 10 Problem 3.


4. Solution: See Linear Algebra Done Right Solution Manual Chapter 10 Problem 4.


5. Solution: Let $V=\C^n$, where $n$ is the number of the rows of $B$. Choose a basis $v_1,\cdots,v_n$ of $V$. Let $T\in\ca L(V)$ so that\[\ca M(T,(v_1,\cdots,v_n))=B.\]By 8.29, there exists another basis $w_1,\cdots,w_n$ of $V$ such that $\ca M(T,(w_1,\cdots,w_n))$ is an upper-triangular matrix.

Let $A=\ca M(I,(w_1,\cdots,w_n),(v_1,\cdots,v_n))$, then $A$ is intertible. It follows from 10.7 that\[\ca M(T,(w_1,\cdots,w_n)=A^{-1}\ca M(T,(v_1,\cdots,v_n))A=A^{-1}BA.\]Since $\ca M(T,(w_1,\cdots,w_n))$ is an upper-triangular matrix, so is $A^{-1}BA$.

See also Linear Algebra Done Right Solution Manual Chapter 10 Problem 5.


6. Solution: Let $V=\R^2$, then $e_1=(1,0)$, $e_2=(0,1)$ is a basis of $V$. Let $T$ be the unique operator in $\ca L(V)$ such that $Te_1=e_2$ and $Te_2=-e_1$. Then \[T^2e_1=Te_2=-e_1\quad \text{and}\quad T^2e_2=-Te_1=-e_2.\]Hence $\ca M(T^2,(e_1,e_2))=-I$. In particular, $\m{trace}~T^2=-2 < 0$.

See also Linear Algebra Done Right Solution Manual Chapter 10 Problem 6.


7. Solution: See Linear Algebra Done Right Solution Manual Chapter 10 Problem 7.


8. Solution: See Linear Algebra Done Right Solution Manual Chapter 10 Problem 8.


9. Solution: See Linear Algebra Done Right Solution Manual Chapter 10 Problem 9.


10. Solution: Choose an orthonormal basis $e_1,\cdots,e_n$ of $V$. By 7.10, we have
\[\ca M(T^*,(e_1,\cdots,e_n))=\overline{\ca M(T,(e_1,\cdots,e_n))}^T.\]By the definition of the trace of a matrix, we have $\m{trace}~A^T=\m{trace}~A$ for any square matrix $A$. Therefore, by 10.16, we have\begin{align*}\m{trace}~T^*=&\m{trace}(\ca M(T^*,(e_1,\cdots,e_n)))\\=&\m{trace}(\overline{\ca M(T,(e_1,\cdots,e_n))}^T)\\=&\m{trace}(\overline{\ca M(T,(e_1,\cdots,e_n))})\\ =&\overline{\m{trace}(\ca M(T,(e_1,\cdots,e_n)))}\\ =&\overline{\m{trace}~T}.\end{align*}

Here we use the fact that $\m{trace}~\overline A=\overline{\m{trace}~A}$ for any square matrix $A$. Why? Prove it.


11. Solution: By 7.35, we know that $T$ is self-adjoint and all the eigenvalues of $T$ are nonnegative. By the Spectral Theorem, $T$ has a diagonal matrix with respect to some orthonormal basis of $V$. Fix this orthonormal basis of $V$. Then $\m{trace}~T=\m{trace}(\ca M(T))=0$.

Note that $\ca M(T)$ is a diagonal matrix with the eigenvalues of $T$ on the diagonal entries and the fact that all the eigenvalues of $T$ are nonnegative, it follows from $\m{trace}(\ca M(T))=0$ that all the eigenvalues of $T$ are zero. Hence $\ca M(T)=0$, which implies $T=0$.


12. Suppose $V$ is an inner product space and $P,Q\in\ca L(V)$ are orthogonal projections. Prove that $\m{trace}(PQ)\geq 0$.


13. Solution: Since $\m{trace}(T)$ is the sum of all eigenvalues and the sum of the diagonal entries of $\ca M(T)$. Hence the third eigenvalue of $T$ is\[51+(-40)+1-(-48)-24=36.\]See also Linear Algebra Done Right Solution Manual Chapter 10 Problem 12.


14. Solution: Choose a basis of $V$. We have $\m{trace}(cT)=\m{trace}(\ca M(cT))$ and $\m{trace}(T)=\m{trace}{\ca M(T)}$ by 10.16. Note that by 3.38, we have $\ca M(cT)=c\ca M(T)$. Therefore, we have\[\m{trace}(\ca M(cT))=\m{trace}(c\ca M(T))=c\m{trace}(\ca M(T)).\]Hence\[\m{trace}(cT)=\m{trace}(\ca M(cT))=c\m{trace}(\ca M(T))=c\m{trace}(T).\]


15. Solution: Choose a basis of $V$. By 10.4, 10.14 and 10.16, we have\begin{align*}&\m{trace}(ST)\\ \text{by 10.16}\quad=&\m{trace}(\ca M(ST))\\ \text{by 10.4}\quad=&\m{trace}(\ca M(S)\ca M(T))\\ \text{by 10.14}\quad=&\m{trace}(\ca M(T)\ca M(S))\\ \text{by 10.4}\quad=&\m{trace}(\ca M(ST))\\ \text{by 10.16}\quad=&\m{trace}(TS).\end{align*}


16. Solution: Let $V=\mb F^2$. Fix a basis $v_1,v_2$ of $V$. Let $S=T$, then $\m{trace}(ST)=\m{trace}(I)=2$ and\[\m{trace}(S)=\m{trace}(T)=2.\]However\[\m{trace}(ST)=2\ne 4=\m{trace}(S)\m{trace}(T).\]See also Linear Algebra Done Right Solution Manual Chapter 10 Problem 14.


17. Solution: We assume that $\dim V=n$. If $T\ne 0$, then $\m{Ker}(T)\ne V$. Let $v_{m+1},\cdots,v_n$ be a basis of $\m{Ker}(T)$, then $m\geq 1$. Extend $v_{m+1},\cdots,v_n$ to a basis $v_1,\dots,v_n$ of $V$. By the proof of 3.22, we have $Tv_1,\cdots,Tv_m$ is linearly independent in $V$. Hence we can extend $Tv_1,\cdots,Tv_m$ to a basis $Tv_1,\cdots,Tv_m,w_{m+1},\cdots,w_m$. Hence there exists $S\in \ca L(V)$ such that $S(Tv_1)=v_1$ (since $m\geq 1$, this is possible), $S(Tv_i)=0$ for all $i=2,\cdots,m$ and $Sw_j=0$ for all $j=m+1,\cdots,n$. Then we have \[STv_1=v_1,\quad STv_i=0,\quad STv_j=S0=0,\quad i=2,\cdots,m,\quad j=m+1,\dots,n.\]Thus $1,0$ are all the eigenvalues of $ST$. Moreover, the eigenvalue $1$ has multiplicity one while the eigenvalue $0$ has multiplicities $n-1$. Since trace is the sum of eigenvalues with mltiplicity, we have $\m{trace}(ST)=1$ and we get a contradiction. Therefore $T=0$.

See also Linear Algebra Done Right Solution Manual Chapter 10 Problem 15.


18. Solution: See Linear Algebra Done Right Solution Manual Chapter 10 Problem 16.


19. Solution: See Linear Algebra Done Right Solution Manual Chapter 10 Problem 18.


20. Solution: See Linear Algebra Done Right Solution Manual Chapter 10 Problem 17.


21. Solution: See Linear Algebra Done Right Solution Manual Chapter 10 Problem 19.


Linearity

This website is supposed to help you study Linear Algebras. Please only read these solutions after thinking about the problems carefully. Do not just copy these solutions.
Close Menu